How to Determine Integration Bounds for Flux Calculation?

Click For Summary
To calculate the flux of the vector field F=4i+2j+3k across the surface S defined by the plane 3x+y+z=4 in the first octant, the integration bounds must be determined by projecting the surface onto the x-y plane. This projection results in the equation y=4-3x, establishing the limits for y from 0 to 4-3x and for x from 0 to 4/3. The integrand simplifies to 17dxdy, indicating that the flux can be computed as 17 times the area of the triangle formed by these bounds. The area of the triangle, with a base of 4/3 and height of 4, can be calculated to finalize the flux value. The discussion highlights the importance of correctly identifying integration limits when evaluating surface integrals.
UrbanXrisis
Messages
1,192
Reaction score
1
Let S be the part of the plane 3x+y+z=4 which lies in the first octant, oriented upward. Find the flux of the vector field F=4i+2j+3k across the surface S.

\int \int F\cdot dS = \int int \left( -P \frac{\partial g}{\partial x} -Q \frac{\partial g}{\partial y} +R \right) dA

\int \int \left( -4(-3)-2(-1)+4-3x-y \right) dA
\int \int \left( 18-3x-y \right) dA

how do I find the ends of integration?
 
Physics news on Phys.org
UrbanXrisis, I really don't understand what you did.

What I would have done is
\int \int F\cdot dS = \int \int \left(F\cdot \hat{n}\right) \left(\frac{dxdy}{\hat{n}\cdot\hat{k}}\right)

where, \hat{n} is the unit normal to the plane. That is, to evaluate the surface integral, I'm simply projecting the surface to the x-y plane and integrating.

Now to find the ends of integration, just project the surface to the x-y plane. In this case, you will get a triangle. From that, you can find the limits of integration
 
UrbanXrisis said:
Let S be the part of the plane 3x+y+z=4 which lies in the first octant, oriented upward. Find the flux of the vector field F=4i+2j+3k across the surface S.

\int \int F\cdot dS = \int int \left( -P \frac{\partial g}{\partial x} -Q \frac{\partial g}{\partial y} +R \right) dA[\quote]
? There were no "P", "Q", "R" in the problem! If you MEAN 3x+ y+ z= 4 so P= 3, y= 1, z= 1 then say that!

\int \int \left( -4(-3)-2(-1)+4-3x-y \right) dA
\int \int \left( 18-3x-y \right) dA
Again mystifying! Assuming you are doing the integration by projecting down into the xy- plane, then d\sigma^{/rightarrow}= (3i+ j+ k)dxdy so the integrand is (4i+ 2j+ 3k) . (3i+ j+ k) dxdy= 17dxdy<br /> <br /> <blockquote data-attributes="" data-quote="" data-source="" class="bbCodeBlock bbCodeBlock--expandable bbCodeBlock--quote js-expandWatch"> <div class="bbCodeBlock-content"> <div class="bbCodeBlock-expandContent js-expandContent "> how do I find the ends of integration? </div> </div> </blockquote> The limits of integration? Projecting the plane, 3x+y+z=4, into the xy-plane gives 3x+ y+ 0= 4 or y= 4- 3x. When y= 0, x= 4/3 so you would integrate with respect to y from 0 to 4- 3x and then with respect to x from 0 to 4/3.<br /> <br /> However, the integral of &#039;17 dA&#039; is just 17A, or 17 times the area of that triangle. What is the area of a triangle with base 4/3 and height 4?
 
I have a similar question to this, with F = 7xi+yj+zk and plane z + 4x + 2y = 12. so what I did was (7xi+ yj+ zk) . (4i+ 2j+ k), is this correct?
 
anyone?
 
Question: A clock's minute hand has length 4 and its hour hand has length 3. What is the distance between the tips at the moment when it is increasing most rapidly?(Putnam Exam Question) Answer: Making assumption that both the hands moves at constant angular velocities, the answer is ## \sqrt{7} .## But don't you think this assumption is somewhat doubtful and wrong?

Similar threads

Replies
3
Views
2K
Replies
20
Views
2K
Replies
2
Views
2K
  • · Replies 7 ·
Replies
7
Views
2K
  • · Replies 6 ·
Replies
6
Views
3K
Replies
12
Views
2K
  • · Replies 8 ·
Replies
8
Views
2K
  • · Replies 7 ·
Replies
7
Views
4K
  • · Replies 3 ·
Replies
3
Views
2K
  • · Replies 4 ·
Replies
4
Views
2K